Saturday, October 15, 2022

Spitzer's Formula

The following identity is an abstract version of the Spitzer's formula introduced in Wendel's 1958 paper [1] and the proof thereof, with some simplifications.

Theorem. Let $\mathcal{A}$ be a unital Banach algebra with the norm $\|\cdot\|$ and unit $1$. Let $\mathbf{P}$ be a bounded linear operator on $\mathcal{A}$ satisfying the following conditions:

  1. $\mathbf{P}$ is a projection, that is, $\mathbf{P}^2 = \mathbf{P}$.
  2. Both $\operatorname{im}\mathbf{P}$ and $\operatorname{im}(\mathbf{I}-\mathbf{P})$ are closed subalgebras of $\mathcal{A}$.
  3. $x$ and $\mathbf{P}x$ commute for any $x \in \mathcal{A}$.
Then for any $a \in \mathcal{A}$ and with the multiplication operator $\mathbf{M}_a$ defined by $\mathbf{M}_a(x) = ax$, we have $$ \sum_{n=0}^{\infty} t^n (\mathbf{P}\mathbf{M}_a)^n 1 = \exp\left(\sum_{n=1}^{\infty} \frac{t^n}{n} \mathbf{P}(a^n) \right) \tag{*} $$ for any scalar $t$ within the radius of convergence of both sides.

Proof. Write $\mathbf{Q} = \mathbf{I} - \mathbf{P}$ for simplicity. We first note that, for any scalars $a_n$'s and $b_n$'s and $x \in \mathcal{A}$, $$ \mathbf{Q} \left[ \sum_{n=0}^{\infty} a_n (\mathbf{P}x)^n \right] = \mathbf{Q}a_0 \qquad\text{and}\qquad \mathbf{P} \left[ \sum_{n=0}^{\infty} b_n (\mathbf{Q}x)^n \right] = \mathbf{P}b_0 \tag{1} $$ provided the respective series converges. Now let $t$ be sufficiently small so that all the series appearing in $\text{(*)}$ converges absolutely, and let $g$ denote the left-hand side of $\text{(*)}$: $$ g = \sum_{n=0}^{\infty} t^n (\mathbf{P}\mathbf{M}_a)^n 1 = (\mathbf{I} - t \mathbf{P}\mathbf{M}_a)^{-1} 1 $$ Hence, $g$ is uniquely determined by the equation $$ (\mathbf{I} - t \mathbf{P}\mathbf{M}_a) g = 1. \tag{2} $$ In light of the above observation, it suffices to prove that the right-hand side of $\text{(*)}$ satisfies the condition $\text{(2)}$. Indeed, let $g^*$ denote the right-hand side of $\text{(*)}$: $$ g^* = \exp\left(\sum_{n=1}^{\infty} \frac{t^n}{n} \mathbf{P}(a^n) \right) = \exp\left(- \mathbf{P} \log(1 - ta) \right) $$ Then we get \begin{align*} (\mathbf{I} - t \mathbf{P}\mathbf{M}_a) g^* &= \mathbf{Q}g^* + \mathbf{P}(\mathbf{I} - t\mathbf{M}_a)g^* \\ &= \mathbf{Q}g^* + \mathbf{P}((1 - ta)g^*) \\ &= \mathbf{Q}\exp\left( -\mathbf{P} \log(1 - ta) \right) + \mathbf{P} \exp\left( \mathbf{Q} \log(1 - ta) \right) \\ &= \mathbf{Q}1 + \mathbf{P}1 \\ &= 1. \end{align*} In the third step, we utilized the fact that $e^x e^y = e^{x+y}$ provided $x$ and $y$ commute. Then $\text{(1)}$ is used in the fourth step. Therefore $g^* = g$. $\square$

References.

[1] Wendel, James G. (1958). "Spitzer's formula: A short proof". Proceedings of the American Mathematical Society. 9 (6): 905–908. doi:10.1090/S0002-9939-1958-0103531-2. MR 0103531.

Sunday, October 2, 2022

Dominated Convergence Theorem for Series

In this post, we will cover a classical but nevertheless powerful result about interchanging the order of limit and sum.

Let $G$ be an abelian group. We will adopt additive notation for the group operation. Assume that $G$ is endowed with a norm $\|\cdot\|$ with respect to which $G$ is complete.

Theorem. Let $(a^{(n)}_k)_{n,k=1}^{\infty} \subseteq G$ and $(M_k)_{k=1}^{\infty} \subseteq [0, \infty)$ satisfy the following conditions:

  1. $\| a^{(n)}_k \| \leq M_k$ holds for each $n$ and $k$;
  2. $(a^{(n)}_k)_{n=1}^{\infty}$ converges in $G$ as $k \to \infty$ for each $n$;
  3. $\sum_{k=1}^{\infty} M_k $ converges.
Then $\sum_{k=1}^{\infty} a^{(n)}_k $ converges for each $n$, and $$ \lim_{n\to\infty} \sum_{k=1}^{\infty} a^{(n)}_k = \sum_{k=1}^{\infty} \lim_{n\to\infty} a^{(n)}_k. $$

Remark. The completeness of $G$ is not necessary provided all the sums appearing in the equality converge in $G$.

Proof. For each $n$, let $S^{(n)} := \sum_{k=1}^{\infty} a^{(n)}_k $. Then for $p \lt q$, we have $$ \left\| \sum_{k=p+1}^{q} a^{(n)}_k \right\| \leq \sum_{k=p+q}^{q} \| a^{(n)}_k\| \leq \sum_{k=p+1}^{q} M_k. $$ Since this bound converges to $0$ as $p, q \to \infty$, the partial sums of $S^{(n)} $ define a Cauchy sequence in $G$ for each $n$. Then the completeness of $G$ ensures that the series $S^{(n)}$ converges in $G$ for each $n$.

Define the sequence $(a_k)_{k=1}^{\infty}$ in $G$ by $a_k := \lim_{n\to\infty} a^{(n)}_k$ for each $k$. Then it is clear that $\| a_k \| \leq M_k$, and so, the sum $S := \sum_{k=1}^{\infty} a_k$ converges by the same reasoning. Finally, for each fixed $p$, \begin{align*} \| S^{(n)} - S \| &\leq \left\| S^{(n)} - \sum_{k=1}^{p} a_k \right\| + \left\| \sum_{k=1}^{p} a_k - S \right\| \\ &\leq \sum_{k=1}^{p} \| a^{(n)}_k - a_k \| + \sum_{k=p+1}^{\infty} \| a^{(n)}_k \| + \sum_{k=p+1}^{\infty} \| a_k \| \\ &\leq \sum_{k=1}^{p} \| a^{(n)}_k - a_k \| + 2 \sum_{k=p+1}^{\infty} M_k. \end{align*} Letting $\limsup$ as $n \to \infty$, we obtain the bound $$ \limsup_{n\to\infty} \| S^{(n)} - S \| \leq 2 \sum_{k=p+1}^{\infty} M_k. $$ Since the left-hand side does not depend on $p$, letting $p \to \infty$ shows that the limsup is in fact $0$, proving $S^{(n)} \to S$ in $G$ as $n \to \infty$. This conclusion is equivalent to the desired assertion, hence the proof is done. $\square$

Example. We show that $$ \sum_{k=1}^{\infty} \frac{2^k}{k} = 0 $$ in $\mathbb{Q}_2$. Indeed, for $n \geq 1$ we have \begin{align*} 0 = \frac{1 - (1 - 2)^{2^n}}{2^n} &= \frac{1}{2^n} \sum_{k=1}^{\infty} (-1)^{k-1} \binom{2^n}{k} 2^k \\ &= \sum_{k=1}^{\infty} (-1)^{k-1} \binom{2^n - 1}{k - 1} \frac{2^k}{k} \\ &= \sum_{k=1}^{\infty} \left[ \prod_{l=1}^{k-1} \left( 1 - \frac{2^n}{l} \right) \right] \frac{2^k}{k}. \end{align*} It is easy to check that the $|\cdot|_2$-norm of the $k$th term is uniformly bounded by $2^{-k}$ for all $n$ and $k$, and $\prod_{l=1}^{k-1} \left( 1 - \frac{2^n}{l} \right) \to 1$ in $\mathbb{Q}_2$ as $n \to \infty$ for each $k$. So by DCT, it follows that $$ 0 = \sum_{k=1}^{\infty} \left[ \lim_{n \to \infty} \prod_{l=1}^{k-1} \left( 1 - \frac{2^n}{l} \right) \right] \frac{2^k}{k} = \sum_{k=1}^{\infty} \frac{2^k}{k}. $$

Remark. The above result can be interpreted in terms of logarithm as follows: Note that the logarithm $\log(z) = \sum_{n=1}^{\infty} \frac{(-1)^{n-1}}{n} z^n$ converges in $\mathbb{Q}_2$ whenever $|z|_2 < 1$, and moreover, $$ \log(1+z) + \log(1+w) = \log((1+z)(1+w)), $$ which initially holds as an identity between formal power series, extends to all $z, w \in \mathbb{Q}_2$ with $|z|_2 < 1$ and $|w|_2 < 1$. So, $$ \sum_{n=1}^{\infty} \frac{2^n}{n} = - \log(1-2) = - \log (-1) = - \frac{1}{2} \log((-1)^2) = -\frac{1}{2}\log 1 = 0. $$

Tuesday, July 26, 2022

Product of two beta distributions

Theorem. Let $p, q, r > 0$. If $X \sim \operatorname{Beta}(p, q)$ and $Y \sim \operatorname{Beta}(p+q, r)$, then $$ XY \sim \operatorname{Beta}(p, q+r). $$

Proof. Let $T_p, T_q, T_r$ be independent random variables such that $T_{\alpha} \sim \operatorname{Gamma}(\alpha)$ for each $\alpha \in \{p, q, r\}$. Then it is well-known that $$ \frac{T_p}{T_p + T_q} \sim \operatorname{Beta}(p, q) \quad\text{and}\quad T_p + T_q \sim \operatorname{Gamma}(p+q). $$ Moreover, these two random variables are independent. Using this, we may realize the joint distribution of $X$ and $Y$ via $$ (X, Y) \stackrel{\text{d}}= \left( \frac{T_p}{T_p + T_q}, \frac{T_p + T_q}{T_p + T_q + T_r} \right). $$ Therefore, we conclude $$ XY \stackrel{\text{d}}= \frac{T_p}{T_p + T_q + T_r} \sim \operatorname{Gamma}(p+q+r) $$ as desired. $\square$

Corollary. Let $p, q > 0$. If $(X_n)_{n\geq 0}$ is a sequence of i.i.d. $\operatorname{Beta}(p, q)$ variables, then $$ \sum_{n=0}^{\infty} (-1)^n X_0 X_1 \cdots X_n \sim \operatorname{Beta}(p, p+q). $$ This is Problem 6524 of American Mathematical Monthly, Vol.93, No.7, Aug.–Sep. 1986.

Proof. Let $S$ denote the sum. By the alternating series estimation theorem, $0 \leq S \leq 1$ holds almost surely. Moreover, if $X\sim \operatorname{Beta}(p, q)$ is independent of $(X_n)_{n\geq 0}$, then $$ S = X_0 \left(1 - \sum_{n=0}^{\infty} (-1)^n X_1 \cdots X_{n+1} \right) \stackrel{\text{d}}= X (1 - S). $$ By noting that this equation uniquely determines all the moments of $S$, we find that this equation has a unique distributional solution by the uniqueness of the Hausdorff moment problem.

Moreover, if $S' \sim \operatorname{Beta}(p, p+q)$ is independent of $X$, then $1-S' \sim \operatorname{Beta}(p+q, p)$. So by the theorem, $$ X(1-S') \sim \operatorname{Beta}(p, p+q) \sim S'.$$ This shows that $S'$ is a distributional solution of the equation, hence by the uniquness, $S \stackrel{\text{d}}= S'$. $\square$

Thursday, July 21, 2022

A simple calculus question

Consider the problem of finding an antiderivative of $$ \int \frac{1}{a + b \cos x} \, \mathrm{d}x, $$ where $|b| \lt a$. It is often done by adopting the Weierstrass substitution $t = \tan(x/2)$. The upshot of this substitution is \begin{align*} \int \frac{1}{a + b \cos x} \, \mathrm{d}x &= \int \frac{1}{a + b\left(\frac{1-t^2}{1+t^2}\right)} \, \frac{2\,\mathrm{d}t}{1+t^2} \\ &= \int \frac{2}{(a+b) + (a-b)t^2} \, \mathrm{d}t \\ &= \frac{2}{\sqrt{a^2-b^2}} \arctan\left(\sqrt{\frac{a-b}{a+b}} \tan \frac{x}{2} \right) + \mathsf{C}. \tag{1} \end{align*} However, there is one caveat in this result. Suppose we want to compute the definite integral $$ \int_{0}^{2\pi} \frac{1}{a + b \cos x} \, \mathrm{d}x. $$ Can we utilize the antiderivative $\text{(1)}$ to find its value? Well, this is not possible, at least directly. This is because the substitution $t = \tan(x/2)$ is discontinuous at every point of the form $$ x = (2k+1)\pi, \qquad k \in \mathbb{Z}. $$ Consequently, the formula $\text{(1)}$ also suffers from discontinuity at those points. The usual trick to avoid this difficulty is to choose another $2\pi$-interval on which $\tan(x/2)$ is continuous. One such choice is the open interval $(-\pi, \pi)$. So, \begin{align*} \int_{0}^{2\pi} \frac{1}{a + b \cos x} \, \mathrm{d}x &= \int_{-\pi}^{\pi} \frac{1}{a + b \cos x} \, \mathrm{d}x \\ &= \lim_{\varepsilon \to 0^+} \int_{-\pi+\varepsilon}^{\pi-\varepsilon} \frac{1}{a + b \cos x} \, \mathrm{d}x \\ &= \lim_{\varepsilon \to 0^+} \left[ \frac{2}{\sqrt{a^2-b^2}} \arctan\left(\sqrt{\frac{a-b}{a+b}} \tan \frac{x}{2} \right) \right]_{-\pi+\varepsilon}^{\pi-\varepsilon} \\ &= \frac{2\pi}{\sqrt{a^2-b^2}}. \end{align*}

So, is this the end of the story? In fact, it turns out that we can find an antiderivative that is vaild on all of $\mathbb{R}$:

Theorem. Let $a, b \in \mathbb{R}$ satisfy $|b| \lt a$. Then \begin{align*} & \int \frac{1}{a + b\cos x} \, \mathrm{d}x \\ &= \frac{1}{\sqrt{a^2 - b^2}} \biggl[ x - 2 \arctan \biggl( \frac{b \sin x}{a + \sqrt{a^2-b^2} + b \cos x} \biggr) \biggr] + \mathsf{C}. \end{align*} This formula is valid on $\mathbb{R}$.

Proof. The Fourier series of the integrand is $$ \frac{1}{a + b\cos x} = \frac{1}{\sqrt{a^2 - b^2}} \left( 1 + 2 \sum_{n=1}^{\infty} r^n \cos(nx) \right), $$ where $r$ is given by $$ r = -\frac{b}{a + \sqrt{a^2 - b^2}} \in (-1, 1). $$ Integrating both sides, we conclude \begin{align*} & \int \frac{1}{a + b\cos x} \, \mathrm{d}x \\ &= \frac{1}{\sqrt{a^2 - b^2}} \left( x + 2 \sum_{n=1}^{\infty} \frac{r^n}{n} \sin(nx) \right) + \mathsf{C} \\ &= \frac{1}{\sqrt{a^2 - b^2}} \left[ x - 2 \operatorname{Im}\left( \log(1 - re^{ix}) \right) \right] + \mathsf{C} \\ &= \frac{1}{\sqrt{a^2 - b^2}} \left[ x + 2 \arctan \left( \frac{r \sin x}{1 - r \cos x} \right) \right] + \mathsf{C}. \end{align*} Plugging the value of $r$ into this, we are done.

Tuesday, July 19, 2022

A nasty integral

In this post, we discuss how certain integrals of rational functions can be computed.

Example 1. $$ \int_{0}^{\infty} \frac{x^{8} - 4x^{6} + 9x^{4} - 5x^{2} + 1}{x^{12} - 10x^{10} + 37x^{8} - 42x^{6} + 26x^{4} - 8x^{2} + 1} \, \mathrm{d}x = \frac{\pi}{2}. $$ This is Problem 11148 of American Mathematical Monthly, Vol.112, April 2005.

Proof. For each $f(z) \in \mathbb{C}[z]$, we define $f^*(z) = \overline{f(\bar{z})}$. This is the same as taking conjugate to the coefficients appearing in $f(z)$. Let \begin{align*} P(z) &= z^8-4 z^6+9 z^4-5 z^2+1, \\ Q(z) &= z^{12}-10 z^{10}+37 z^8-42 z^6+26 z^4-8 z^2+1. \end{align*} The key observation is that all the zeros of the polynomial $$ q(z) = z^3-(2+i) z^2-(1-i) z+1 $$ lies in the upper half-plane $\mathbb{H} = \{ z \in \mathbb{C} : \operatorname{Im}(z) \gt 0\}$, and $$ Q(z) = q(z)q^*(z)q(-z)q^*(-z) $$ is a factorization of $Q(z)$ into coprime factors. Now let $p(z)$ be the unique polynomial such that $\deg p \lt \deg q$ and $$ P(z) \equiv p(z) q^*(z)q(-z)q^*(-z) \pmod{q(z)}. $$ Such $p(z)$ exists because $q(z)$ and $q^*(z)q(-z)q^*(-z)$ are coprime. Also, such $p(z)$ can be computed using the extended Euclidean algorithm. After a tedious computation, it follows that $$ p(z) = \frac{-i z^2-(1-2 i) z+1}{4}. $$ Then by the symmetry and the uniqueness of partial fraction decomposition, it follows that $$ \frac{P(z)}{Q(z)} = \frac{p(z)}{q(z)} + \frac{p^*(z)}{q^*(z)} + \frac{p(-z)}{q(-z)} + \frac{p^*(-z)}{q^*(-z)}. $$ Moreover, using the fact that all the zeros of $q(z)$ lies in $\mathbb{H}$, we can invoke Cauchy's integral theorem to write \begin{align*} \operatorname{PV}\!\! \int_{-\infty}^{\infty} \frac{x^n}{q(x)} \, \mathrm{d}x &= \lim_{R\to\infty} \int_{-R}^{R} \frac{x^n}{q(x)} \, \mathrm{d}x \\ &= \lim_{R\to\infty} \int_{-\pi}^{0} \frac{i (Re^{i\theta})^{n+1}}{q(Re^{i\theta})} \, \mathrm{d}\theta \\ &= \begin{cases} 0, & n \lt \deg q - 1, \\ \pi i, & n = \deg q - 1. \end{cases} \end{align*} Therefore we conclude that $$ \int_{-\infty}^{\infty} \frac{P(x)}{Q(x)} \, \mathrm{d}x = 4 \operatorname{Re}\left[ \operatorname{PV}\!\! \int_{-\infty}^{\infty} \frac{p(x)}{q(x)} \, \mathrm{d}x \right] = 4 \pi i \left( [z^2] p(z) \right) = \pi. $$ The answer is half of the above integral, hence the claim follows. $\square$

Here is another example.

Example 2. $$ \int_{0}^{\infty} \frac{x^{14} - 15x^{12} + 82x^{10} - 190x^{8} + 184x^{6} - 60x^{4} + 16x^{2}}{x^{16} - 20x^{14} + 156x^{12} - 616x^{10} + 1388x^{8} - 1792x^{6} + 1152x^{4} - 224x^{2} + 16} \, \mathrm{d}x = \frac{\pi}{2}. $$

Proof. Apply the same argument as above with the choices $$ q(z) = z^4+(3-i) z^3-(1+3 i) z^2-(6+2 i) z-2 $$ and $$ p(z) = \frac{-i z^3-3 i z^2-2 i z}{4}. $$ We assure the reader that all the zeros of $q(z)$ lie in the upper half-plane, and the denominator of the integral admits the same form of factorization into coprime factors as before. $\square$

Sunday, July 10, 2022

Orthogonal decomposition using Gram determinants

Theorem. Let $\mathcal{H}$ be a Hilbert space equipped with the inner product $\langle \cdot, \cdot \rangle$, and let $w, v_1, \ldots, v_n \in \mathcal{H}$. If $$ G(w_1, \ldots, w_n) = \det [\langle w_i, w_j \rangle]_{i,j=1}^{n} $$ denotes the Gram determinant, then the followings hold true:

  1. The vector $w_{\perp}$ defined by the (formal) determinant $$ w_{\perp} = \frac{1}{G(v_1,\ldots,v_n)}\begin{vmatrix} w & v_1 & \cdots & v_n \\ \langle v_1, w \rangle & \langle v_1, v_1 \rangle & \cdots & \langle v_1, v_n \rangle \\ \vdots & \vdots & \ddots & \vdots \\ \langle v_n, w \rangle & \langle v_n, v_1 \rangle & \cdots & \langle v_n, v_n \rangle \end{vmatrix} \tag{1} $$ is orthogonal to $V = \operatorname{span}\{v_1, \ldots, v_n\}$, and in fact, $$ w = w_{\perp} + (w - w_{\perp}) $$ is the orthogonal decomposition of $w$.
  2. The square-distance between $w$ and $V$ is given by $$ \|w_{\perp}\|^2 = \operatorname{dist}(w, V)^2 = \frac{G(w, v_1, \ldots, v_n)}{G(v_1, \ldots, v_n)}. \tag{2} $$

Proof. Extend the notation by letting $$ G(\{v_i\}_{i=1}^{n}, \{w_i\}_{i=1}^{n}) = \det [\langle v_i, w_j \rangle]_{i,j=1}^{n}, $$ and then define the linear functional $\ell$ on $\mathcal{H}$ by $$ \ell(u) = G(\{u, v_1, \ldots, v_n\}, \{w, v_1, \ldots, v_n\}). $$ By the Riesz representation theorem, there exists $h \in \mathcal{H}$ such that $\ell(u) = \langle h, u \rangle$ for all $u \in \mathcal{H}$. Since $\ell(v_i) = 0$ for all $i = 1, \ldots, n$, it follows that $h$ is orthogonal to $V$. Moreover, expanding the determinant defining $\ell(u)$ along the first line, we see that \begin{align*} \ell(u) &= G(v_1, \ldots, v_n) \langle u, w \rangle \\ &\quad + \sum_{i=1}^{n} (-1)^i G(\{w, v_1, \ldots, \widehat{v_i}, \ldots, v_n\}, \{v_1, \ldots, v_n\}) \langle u, v_i \rangle, \end{align*} and so, \begin{align*} h &= G(v_1, \ldots, v_n) w \\ &\quad + \sum_{i=1}^{n} (-1)^i G(\{w, v_1, \ldots, \widehat{v_i}, \ldots, v_n\}, \{v_1, \ldots, v_n\}) v_1, \end{align*} which is precisely the formal determant in $\text{(1)}$. This also implies that $h$ is a linear combination of $w, v_1, \ldots, v_n$ with the coefficient of $w$ given by $G(v_1, \ldots, v_n)$, hence $$ w_{\perp} = \frac{h}{G(v_1, \ldots, v_n)} \in w + V. $$ This and $w_{\perp} \perp V$ together proves the first item of the theorem. For the second item, the equality $\|w_{\perp}\| = \operatorname{dist}(w, V)$ is obvious by the orthogonality. Moreover, $$ \|w_{\perp}\|^2 = \langle w_{\perp}, w_{\perp} \rangle = \langle w_{\perp}, w \rangle = \frac{\ell(w)}{G(v_1, \ldots, v_n)} = \frac{G(w, v_1, \ldots, v_n)}{G(v_1, \ldots, v_n)}. $$ This completes the proof. $\square$

Friday, June 10, 2022

A tricky convergence problem

This is a slight simplification of the Math.SE user @Mike's wonderful solution [1] that is rephrased in my own words.

Theorem. Suppose

  1. $(a_n)_{n\geq 1}$ is a non-negative, non-increasing sequence,
  2. $(f(n))_{n\geq 1}$ is a sequence of positive integers such that $f(n) \to \infty$, and
  3. $\sum_{n\geq 1} a_{f(n)} $ converges.
Then $\sum_{n\geq 1} \frac{a_n}{f(n)}$ also converges.

Proof. We partition $\mathbb{N}_1 = \{1, 2, \ldots\}$ into two subsets $A$ and $B$, where \begin{align*} A &= \{ n \in \mathbb{N}_1 : f(n) > n \}, \\ B &= \{ n \in \mathbb{N}_1 : f(n) \leq n \}. \end{align*} Then it suffices to show that both $ \sum_{n\in A} \frac{a_n}{f(n)} $ and $ \sum_{n\in B} \frac{a_n}{f(n)} $ are finite.

First sum. Since $(a_n)_{n\in\mathbb{N}_1}$ is non-increasing and $1 \leq f(n) \leq n$ for each $n \in B$, we get $$ \sum_{n\in B} \frac{a_n}{f(n)} \leq \sum_{n \in B} a_{f(n)} \lt \infty. $$

Second sum. Now enumerate $f(\mathbb{N}_1)$ as $\{k_1 \lt k_2 \lt \ldots \}$, and define $$A_i = A \cap [k_i, k_{i+1})$$ for each $i$. Then the following implication holds true: \begin{align*} n \in A_i \quad\implies\quad f(n) \gt n \geq k_i \quad\implies\quad f(n) \geq k_{i+1}. \end{align*} So it follows that $$ \sum_{n \in A_i} \frac{a_n}{f(n)} \leq \sum_{n \in A_i} \frac{a_{k_i}}{k_{i+1}} = \frac{k_{i+1} - k_i}{k_{i+1}} a_{k_i} \leq a_{k_i}. $$ Summing both sides over $i$, \begin{align*} \sum_{n \in A} \frac{a_n}{f(n)} &\leq \sum_{i=1}^{\infty} a_{k_i} \\ &\leq \sum_{i=1}^{\infty} \sum_{n \mathbin{:} f(n) = k_i} a_{f(n)} \\ &= \sum_{n=1}^{\infty} a_{f(n)} \lt \infty. \end{align*} Therefore the desired conclusion follows.


References.
  1. [1] Mike, If $\sum_{n=1}^{\infty}{a_{[f(n)]}}$ converges, then $\sum_{n=1}^{\infty}{\frac{a_n}{f(n)}}$ converges., URL (version: 2022-06-05): https://math.stackexchange.com/q/4464970

A property of an integrable function on $\mathbb{R}$

Theorem. Let $f \in L^1(\mathbb{R}, \mathrm{d}x)$. Then $$ \lim_{n\to\infty} f(nx) = 0 \quad \text{for a.e. } x. $$

The proof below is essentially what is shown in [1], written in my own words.

Proof. Fix $\varepsilon \gt 0$ and define $E = \{x \in \mathbb{R} : |f(x)| \geq \varepsilon\}$. Then for any $[a, b] \subseteq (0, \infty)$, \begin{align*} \int_{a}^{b} \sum_{n=1}^{\infty} \mathbf{1}_{E}(nx) \, \mathrm{d}x &= \sum_{n=1}^{\infty} \frac{1}{n} \int_{na}^{nb} \mathbf{1}_{E}(t) \, \mathrm{d}t \\ &= \int_{E} \sum_{n=1}^{\infty} \frac{1}{n} \mathbf{1}_{\{na \leq t \leq nb\}} \, \mathrm{d}t \\ &= \int_{E} \sum_{n=1}^{\infty} \frac{1}{n} \mathbf{1}_{\{ t/b \leq n \leq t/a \}} \, \mathrm{d}t \\ &\leq \int_{E} (2 + \log(b/a)) \, \mathrm{d}t \\ &= (2 + \log(b/a)) |E|, \end{align*} where $|E|$ denotes the Lebesgue measure of $E$. Since $f$ is integrable, $|E| \lt \infty$. So the sum $\sum_{n=1}^{\infty} \mathbf{1}_{E}(nx)$ is finite and hence $nx \in E$ for only finitely many $n$ for a.e. $x$ in $[a, b]$. Since $[a, b]$ is arbitrary, the same is true for a.e. $x$ in $\mathbb{R}$. From this, it is now routine to conclude the desired claim. $\square$


References
  • [1] Emmanuel Lesigne. On the behavior at infinity of an integrable function. American Mathematical Monthly, Mathematical Association of America, 2010, 117 (2), pp.175-181. ⟨hal-00276738v3⟩

Tuesday, June 7, 2022

A simple but useful estimate

Theorem. Let $f : \mathbb{R} \to \mathbb{R}$ be a continuous, integrable function of bounded variation. Then $\sum_{n\in\mathbb{Z}} f(n)$ converges absolutely, and we have $$ \left| \sum_{n\in \mathbb{Z}} f(n) - \int_{\mathbb{R}} f(x) \, \mathrm{d}x \right| \leq \frac{1}{2} V(f), $$ where $V(f) = \int_{\mathbb{R}} |\mathrm{d}f(x)|$ denotes the total variation of $f$ over $\mathbb{R}$.

Proof. Let $\tilde{B}(x) = x - \lfloor x \rfloor - \frac{1}{2}$. Then performing integration by parts, for $a \le b$, \begin{align*} \int_{a}^{b} f(x) \, \mathrm{d}x - \sum_{n \in [a, b]\cap\mathbb{Z}} f(n) &= \int_{[a, b]} f(x) \, \mathrm{d}\tilde{B}(x) \\ &= [f(x)\tilde{B}(x)]_{a^-}^{b} - \int_{[a,b]} \tilde{B}(x) \, \mathrm{d}f(x). \end{align*} Using the fact that $|\tilde{B}(x)| \leq \frac{1}{2}$, it therefore follows that \begin{align*} \left| \int_{a}^{b} f(x) \, \mathrm{d}x - \sum_{n \in [a, b]\cap\mathbb{Z}} f(n) \right| \leq \frac{|f(a)| + |f(b)|}{2} + \frac{1}{2} \int_{[a,b]} \, |\mathrm{d}f(x)| \end{align*} From the assumption, it is easy to check that $f(x) \to 0$ as $|x| \to \infty$. So, the above estimate shows that $\sum_{n\in\mathbb{Z}} f(n)$ satisfies the Cauchy criterion and hence converges as $a \to -\infty$ and $b \to \infty$. Moreover, noting that $|f|$ also satisfies the assumption in plcae of $f$, we find that this sum converges absolutely. Finally, passing to the limit as $[a,b] \uparrow \mathbb{R}$ proves the desired bound. $\square$

Cauchy–Binet formula

Write $[n]$ for the set $\{1,2,\ldots,n\}$ and $\binom{[n]}{k}$ for the family of all subets of $[n]$ and size $k$. Also, for an $m \times n$ matrix $X$, $I \subseteq [m]$, and $J \subseteq [n]$, let $X_{I,J}$ denote the submatrix obtained from $X$ by deleting row $i$ for each $i \notin I$ and column $j$ for each $j\notin [J]$.

Theorem. (Cauchy–Binet formula) Let $A$ be a $k\times n$ matrix and $B$ be an $n\times k$ matrix over the field $F$. Then $$\det(AB) = \sum_{S\subseteq\binom{[n]}{k}} \det(A_{[k],S}B_{S,[k]}).$$

1st Proof. Let $(e_1,\ldots, e_m)$ denote the standard basis of $F^m$. (For convenience, we will abuse the notation so $e_i$ stands for the $i$th standard basis vector in any of $F^m$, $m \geq i$.) Then \begin{align*} \det(AB)(e_1 \wedge \cdots \wedge e_k) &= \bigl( {\textstyle\bigwedge}^k AB \bigr)(e_1 \wedge \cdots \wedge e_k) \\ &= \bigl( {\textstyle\bigwedge}^k A \bigr)[(Be_1) \wedge \cdots \wedge (Be_k)]. \end{align*} Now by noting that $Be_j = \sum_{i=1}^{n} B_{ij} e_i$ is the $j$th column of $B$, we may expand the exterior product in the last line as \begin{align*} &[(Be_1) \wedge \cdots \wedge (Be_k)] \\ &= \sum_{\substack{\rho : [k] \to [n] \\ \sigma \text{ injective} }} (B_{\rho(1),1} e_{\rho(1)}) \wedge \cdots \wedge (B_{\rho(k),k} e_{\rho(k)} ) \\ &= \sum_{\substack{\sigma : [k] \to [n] \\ \sigma \text{ increasing} }} \sum_{\substack{\tau : [k] \to [k] \\ \tau \text{ bijective}}} \left( \prod_{i=1}^{k} B_{\sigma(\tau(i)),i} \right) (e_{\sigma(\tau(1))} \wedge \cdots \wedge e_{\sigma(\tau(k))} ) \\ &= \sum_{\substack{\sigma : [k] \to [n] \\ \sigma \text{ increasing} }} \sum_{\substack{\tau : [k] \to [k] \\ \tau \text{ bijective}}} \operatorname{sgn}(\tau) \left( \prod_{i=1}^{k} B_{\sigma(\tau(i)),i} \right) (e_{\sigma(1)} \wedge \cdots \wedge e_{\sigma(k)} ) \\ &= \sum_{\substack{\sigma : [k] \to [n] \\ \sigma \text{ increasing} }} \det(B_{\sigma([k]),[k]}) (e_{\sigma(1)} \wedge \cdots \wedge e_{\sigma(k)} ) \end{align*} Plugginb this back, we therefore get \begin{align*} &\det(AB)(e_1 \wedge \cdots \wedge e_k) \\ &= \sum_{\substack{\sigma : [k] \to [n] \\ \sigma \text{ increasing} }} \det(B_{\sigma([k]),[k]}) [(Ae_{\sigma(1)}) \wedge \cdots \wedge (Ae_{\sigma(k)}) ] \\ &= \sum_{\substack{\sigma : [k] \to [n] \\ \sigma \text{ increasing} }} \det(B_{\sigma([k]),[k]}) \det(A_{[k],\sigma([k])}) (e_1 \wedge \cdots \wedge e_k) \\ &= \sum_{S \in \binom{[n]}{k}} \det(A_{[k],S}) \det(B_{S,[k]}) (e_1 \wedge \cdots \wedge e_k). \end{align*} This completes the proof. $\square$

2nd Proof. We specialize in the case $F = \mathbb{C}$. Also, for each index set $I$, we abbreviate $\wedge_{i\in I} e_i = \wedge I$. Then using inner product between multivectors, we get \begin{align*} \det(AB) &= \left\langle (\wedge^k A^*)(\wedge [k]), (\wedge^k B)(\wedge [k]) \right\rangle \\ &= \sum_{S \in \binom{[n]}{k}} \left\langle (\wedge^k A^*)(\wedge [k]), \wedge S \right\rangle \left\langle \wedge S, (\wedge^k B)(\wedge [k]) \right\rangle \\ &= \sum_{S \in \binom{[n]}{k}} \det(A_{[k],S})\det(B_{S,[k]}) \end{align*}

A similar technique shows:

Proposition. (Coefficients of a characteristic Polynomial) Let $A$ be an $n\times n$ matrix. Then $$\det(zI_n + A) = \sum_{k=0}^{n} z^{n-k} \sum_{S\subseteq\binom{[n]}{k}} \det(A_{S,S}).$$

Proposition. Let $a_1, \ldots, a_p \in \mathbb{C}$ and $v_1, \ldots, v_p \in \mathbb{C}^n$. Then $$\det\left( \sum_{j=1}^{p} a_j v_j v_j^{*} \right) = \sum_{\{j_1 \lt \ldots \lt j_n \} \subseteq [p]} (a_{j_1} \cdots a_{j_n}) \left| \det( v_{j_1}, \ldots, v_{j_n} ) \right|^2. $$

Determinant of a Cauchy matrix

Throughout this posting, the expressions $$ \sum_{\mathit{vars} \mathop{:} \mathit{conds}} f(\mathit{vars}) \qquad\text{and}\qquad \prod_{\mathit{vars} \mathop{:} \mathit{conds}} f(\mathit{vars}) $$ will refer to the sum and product of $f(\mathit{vars})$ over all possible values of $\mathit{vars}$ subject to the constraints $\mathit{conds}$, respectively.

Theorem. (Determinant of a Cauchy matrix) Let $A$ be an $n\times n$ matrix with elements $a_{ij}$ in the form $$ a_{ij} = \frac{1}{x_i + y_j}. $$ (Think of $A$ as an matrix over the field of rational functions in variables $x_i$'s and $y_j$'s.) Then $$ \det A = \frac{\prod_{i, j \mathbin{:} i \lt j} (x_i - x_j)(y_i - y_j)}{\prod_{i, j} (x_i + y_j)}. $$

Proof. We prove the claim by induction on $n$. The base case is clearly true, so it suffices to establish the inductive step.

Suppose that the claim holds for all $(n-1)\times(n-1)$ matrices. Also, recall that if $p(z)$ is a polynomial of degree less than $n$ and if $\alpha_1, \ldots, \alpha_n$ are distinct, then we have the partial fraction decomposition $$ \frac{p(z)}{\prod_{j} (z - \alpha_j)} = \sum_{k} \frac{p(\alpha_k)}{(z - \alpha_k) \prod_{j \mathbin{:} j \neq k} (\alpha_k - \alpha_j)}. $$ So, by regarding $x_n$ as the variable and applying the partial fraction decomposition, \begin{align*} &\frac{\prod_{i \mathbin{:} i \lt n} (x_i - x_n)}{\prod_{j} (x_n + y_j)} \\ &= \sum_{k} \frac{\prod_{i \mathbin{:} i \lt n} (x_i + y_k)}{(x_n + y_k) \prod_{j \mathbin{:} j \neq k} (y_j - y_k)} \\ &= \sum_{k} (-1)^{n-k} \frac{\prod_{i \mathbin{:} i \lt n} (x_i + y_k)}{(x_n + y_k) \bigl[ \prod_{j \mathbin{:} j \lt k} (y_j - y_k) \bigr]\bigl[ \prod_{j \mathbin{:} k \lt j} (y_k - y_j) \bigr]} \end{align*} From this, it follows that \begin{align*} &\frac{\prod_{i, j \mathbin{:} i \lt j} (x_i - x_j)(y_i - y_j)}{\prod_{i, j} (x_i + y_j)} \\ &= \frac{\bigl[ \prod_{i, j \mathbin{:} i \lt j \lt n} (x_i - x_j) \bigr]\bigl[ \prod_{i, j \mathbin{:} i \lt j} (y_i - y_j) \bigr]}{\prod_{i, j \mathbin{:} i \lt n} (x_i + y_j)} \cdot \frac{\prod_{i \mathbin{:} i \lt n} (x_i - x_n)}{\prod_{j} (x_n + y_j)} \\ &= \sum_{k} \frac{(-1)^{n-k}}{x_n + y_k} \cdot \frac{\bigl[ \prod_{i, j \mathbin{:} i \lt j \lt n} (x_i - x_j) \bigr]\bigl[ \prod_{i, j \mathbin{:} i \lt j; i \neq k; j \neq k} (y_i - y_j) \bigr]}{\prod_{i, j \mathbin{:} i \lt n; j\neq k} (x_i + y_j)} \end{align*} However, the last line is precisely the cofactor expansion of $\det A$ along the $n$th row, where the determinants of the submatrices are computed using the inductive hypothesis. Therefore the inductive step is established and the proof is done. $\square$

Sunday, January 2, 2022

Dominated Convergence Theorem Revisited

Theorem. Let $(X, \mathcal{F}, \mu)$ be a measure space, $f_n : X \to [0, \infty]$ a sequence of measurable functions converging almost everywhere to a measurable function $f$ so that $f_n \leq f$ for all $n$. Then $$ \lim_{n\to\infty} \int_{X} f_n \, \mathrm{d}\mu = \int_{X} f \, \mathrm{d}\mu. $$

Note that $f$ need not be integrable and $(f_n)$ need not be monotone increasing.

Proof. By the Fatou's lemma, $$ \int_{X} f \, \mathrm{d}\mu = \int_{X} \varliminf_{n\to\infty} f_n \, \mathrm{d}\mu \leq \varliminf_{n\to\infty} \int_{X} f_n \, \mathrm{d}\mu \leq \varlimsup_{n\to\infty} \int_{X} f_n \, \mathrm{d}\mu \leq \int_{X} f \, \mathrm{d}\mu.$$